Inscription / Connexion Nouveau Sujet
Niveau Licence Maths 1e ann
Partager :

Serie numerique

Posté par
astroq123
27-05-17 à 13:58

Salut, pouvez vous m'aider svp:
Je cherches a etudier la serie de terme general \frac{(-1)^n}{ln(n)+cos(n)}

J'ai essayé de faire un developpement asymptotique, mais ca ne marche pas,
Avez vous des idees?
Merci

Posté par
Glapion Moderateur
re : Serie numerique 27-05-17 à 14:17

Salut, utilise le critère des séries alternées (si le terme général des valeurs absolues décroit et tend vers 0 alors la série converge).

Posté par
astroq123
re : Serie numerique 27-05-17 à 14:29

\frac{1}{ln(n)+cos(n)} n'est pas decroissante !! Vous pouvez tracer le graphe sur une calculatrice programmée pour le voir

Posté par
Glapion Moderateur
re : Serie numerique 27-05-17 à 14:43

Majore le terme général par 1/(ln(n) -1) par exemple et là ça sera décroissant.

Posté par
astroq123
re : Serie numerique 27-05-17 à 15:02

Le terne general n'a pas un signe cst,
Aussi la majoration n'a pas un sens là

Posté par
etniopal
re : Serie numerique 27-05-17 à 15:45

La suite u  définie par  u(n) = \frac{1}{ln(n) + cos(n)} converge vers 0 .

La nature de la série de terme général (sgtg en abrégé ) (-1)nu(n)  est de même nature que la sdtg   v(n) := u(2n)  - u(2n+1)   .
On a  : v(n)       1/(ln(2n)  + 1) - 1/(ln(2n+1) - 1) )   et ( sauf erreur )  
1/(ln(2n)  + 1) - 1/(ln(2n+1) - 1) ) \sim 1/2n(ln(n))² .

Reste à voir si   \int_{12}^{{+\infty}}{\frac{1}{x(ln(x))²}}dx < +\infty   ou pas .

Posté par
etniopal
re : Serie numerique 27-05-17 à 15:55

Je crois qu'il vaut mieux  majorer  v(n) par  w(n) := 1/(ln(2n)  - 1) - 1/(ln(2n+1) +1) ) et trouver un équivalent simple de w(n)  .

Posté par
Glapion Moderateur
re : Serie numerique 27-05-17 à 18:00

Citation :
Le terne general n'a pas un signe cst,

ben si, il est toujours positif.

Posté par
astroq123
re : Serie numerique 27-05-17 à 18:04

Oui, mais est ce qu'elle est decroissante tout le temps?! (Tracez le graphe)

Posté par
Alexique
re : Serie numerique 27-05-17 à 21:52

@Glapion @etnopial
J'ai l'impression que vous cherchez à appliquer des théorèmes de comparaisons de séries à termes positifs...à des séries qui ne sont pas à termes positifs... En plus, Il vaw(n)
 \\  \sim \dfrac{2}{\ln(n)^2}, dont la série associée diverge. Peut-être va-t-il falloir se faire à l'idée que la série diverge et évaluer plus précisément ses sommes partielles...

Posté par
Alexique
re : Serie numerique 27-05-17 à 21:53

zut : w(n) \sim \dfrac{2}{\ln(n)^2}

Posté par
luzak
re : Serie numerique 28-05-17 à 10:04

Bonjour !
J'essaie ça...
u_n=\dfrac{(-1)^n}{\ln n+\cos n}=a_n+b_n,\;a_n=\dfrac{(-1)^n}{\ln n}

b_n=\dfrac{-\cos n}{\ln n(\ln n+\cos n)}=v_n+w_n,\;v_n=\dfrac{-\cos n}{(\ln n)^2}

w_n=\dfrac{\cos^2n}{(\ln n)^2(\ln n+\cos n)}

w_n\geqslant\dfrac{\cos^2n}{(\ln n)^2(\ln n+1)}=\dfrac{1+\cos(2n)}{2(\ln n)^2(\ln n+1)}\geqslant0

Par théorème d'Abel les séries \sum\dfrac{\cos(2n)}{2(\ln n)^2(\ln n+1)},\;\sum v_n,\;\sum a_n sont convergentes.
La série à termes positifs \sum\dfrac{1}{2(\ln n)^2(\ln n+1)} est divergente (comparaison avec \sum 1/n).

Il semble donc que la série \sum u_n est divergente.

Posté par
astroq123
re : Serie numerique 28-05-17 à 13:27

Je pense qu'il faut faire un developpement asymptotique, apres avoir se debarraser de (-1)^n pour mettre le reste de ce developpement, puisqu'il n'est pas absolument convergent, avec un terme qui lui ressemble et qui soit de signe cst, pour chercher un "equivalent"

Posté par
Alexique
re : Serie numerique 28-05-17 à 15:23

@luzak : bravo ! Je m'etais découragé en voyant b_n sans me douter que l'iteration suivante w_n serait convainquante. L'utilisation d'Abel me semblait inevitable pour les séries en cosinus.

Posté par
etniopal
re : Serie numerique 28-05-17 à 16:53

luzak   bonjour !
   Ton bn =  un - an = (-1)n+1cos(n)/ln(n)(ln(n) + cos(n))
puis  si vn = (-1)n+1cos(n)/ln²(n)
wn = bn -  vn = (-1)n+1cos²(n)/ln²(n)(ln(n) + cos(n))
.....
Il y a toujours un    (-1) n en facteur et je ne vois pas arriver de série à termes positifs .

D'ailleurs cette façon de faire consiste à trouver un DA de   n 1/(1 + cos(n)/ln(n))   à partir du DL de  t   1/(1 + t)   en 0.

Posté par
luzak
re : Serie numerique 28-05-17 à 17:15

Bonsoir etniopal
Effectivement j'ai "perdu" un (-1)^n au passage ! Merci de ta vigilance !
Tout est à refaire...

Posté par
etniopal
re : Serie numerique 28-05-17 à 17:36

En  posant un = (-1)n/(ln(n) + cos(n))  et sn =  u2n  + u2n    ce que j'avais fait ne conduit à rien car  on ne peut pas dire que  les sn ont le même signe ( même àpcr )

On a  :  sn =   tn +   t'n  où

   tn  \1/2nln²(2n)     et     t'n  = (cos(2n+1) - cos(2n))/(ln(2n) + cos(2n))(ln(2n+1) + cos(2n+1))

La sdtg     tn   converge  .
En remplaçant   (cos(2n+1) - cos(2n)) par -2sin(2n + 1/2)sin(1/2) on pourrait peut être  caser de  l'Abel ?

Posté par
etniopal
re : Serie numerique 28-05-17 à 17:38

sn = u2n  + u2n+1  

Posté par
etniopal
re : Serie numerique 28-05-17 à 18:05

Autre correction :     C'est    tn   \sim  1/2nln²(2n)  au lieu de   tn  \1/2nln²(2n)

Posté par
Alexique
re : Serie numerique 29-05-17 à 01:11

Bonsoir,

j'ai fait suivre ici   également des fois que des bonnes idées sur un site inspireraient les forumeurs d'un autre... Bonne recherche

Posté par
luzak
re : Serie numerique 30-05-17 à 14:31

Pour n\geqslant 4 (question d'avoir \ln n>1) on a u_n=\dfrac{(-1)^n}{\ln n+\cos n}=\dfrac{(-1)^n}{\ln n}\sum_{p\geqslant0}\dfrac{(-\cos n)^p}{(\ln n)^p}.

Soit x_{n,p}=\dfrac{(-1)^{n+p}\cos^pn}{(\ln n)^{p+1}}.
En fixant p, x_{n,p}=a_nb_n,\;a_n=(-1)^p(-1)^n\cos^pn,\;b_n=(\ln n)^{-1-p}).

On a aussi (-1)^pa_n=\Re{(-1)^ne^{inp}}=\Re( e^{i(p+\varepsilon)n}) (\varepsilon\in\{-1,1\}).

Dans un premier temps j'avais mis (-e^{ip})^n mais le majorant \dfrac{1}{\cos(p/2)} s'avère ingérable puisque 0 est une valeur d'adhérence de la suite p\mapsto\cos(p/2)

Dans le but d'utiliser une sommation d'Abel, soit A_N=\sum_{4\leqslant n<N}a_n partie réelle de (-1)^p\sum_{4\leqslant n<N}e^{i(p+\varepsilon)n}.

De manière classique : |A_N|\leqslant\dfrac{1}{\sin\frac{|p+\varepsilon|}2}.

Il se trouve que \sin(p+1)/2,\;\sin(p-1)/2 ne peuvent être simultanément proches de 0 :

En effet, si \sin\dfrac{p-1}{2}\leqslant\dfrac{\sin 1}{2} on a \dfrac{p-1}{2}=k\pi+v,\;0<v<\pi/2 et \dfrac{p+1}{2}=k\pi+v+1 donc |\sin\dfrac{p+1}{2}|=\sin v\cos1+\sin1\cos v\geqslant\sin1\cos v\geqslant\dfrac{\sin1}{2}.

On a donc toujours la majoration |A_N|\leqslant M=\dfrac2{\sin1}

De manière classique (théorème d'Abel), la suite n\mapsto b_n étant décroissante, de limite 0 (la limite est même uniforme par rapport à p), on a aussi :
la série \sum a_nb_n convergente (majoration des tranches de Cauchy) et la somme s_p=\sum_{n\geqslant4}x_{n,p} vérifie |s_p|\leqslant\dfrac{M}{(\ln4)^{p+1}}.

Par interversion de limites dans une série uniformément convergente :

\lim_{P\to +\infty}\sum_{0\leqslant p\leqslant P}s_p=\lim_{P\to +\infty}\Bigl(\sum_{0\leqslant p\leqslant P}\Bigl(\lim_{N\to +\infty}\sum_{4\leqslant n<N}x_{n,p}\Bigr)\Bigr)=\lim_{N\to +\infty}\Bigl(\lim_{0\leqslant p\leqslant P}\sum_{4\leqslant n<N}x_{n,p}\Bigr)=\lim_{N\to +\infty}\sum_{4\leqslant n<N}u_n

En espérant que c'est exempt de "plantage"...

Posté par
luzak
re : Serie numerique 30-05-17 à 15:14

Désolé : il y a gros plantage...

Mes \dfrac{p\pm1}2 sont en fait des \dfrac{p\pm\pi}2, les sinus ont les mêmes valeurs absolues et le majorant M est toujours \dfrac1{|\cos(p/2)|}.

Après ça ne va plus pour la convergence uniforme, dommage !

Posté par
Alexique
re : Serie numerique 30-05-17 à 16:10

Apparemment, traité ici, j'essaierai de me convaincre plus tard !

Posté par
luzak
re : Serie numerique 31-05-17 à 10:39

Bonjour Alexique et merci pour le lien !
J'ai essayé de le voir  :
Il procède comme moi avec une série double et note M_p ce que j'ai appelé M=\dfrac1{|\cos(p/2)|} sauf qu'il a une forme plus compliquée du genre \sum_{0\leqslant r\leqslant p}\dfrac2{1-\cos(r+\pi)} (je sais comment il arrive à cette complication, mais c'est sans grande importance).

Après , ça se corse : il faut utiliser l'existence d'un réel \mu tel que |\pi-\dfrac pq|\leqslant\dfrac1{q^{\mu+\varepsilon}} pour tout (p,q,\varepsilon)\in\N\times\N\times\R_+^*,\;q assez grand.
(notion de "mesure" d'un irrationnel ou "constante de Liouville").
De là il prétend qu'on doit pouvoir majorer M_p par une "puissance de p" et conclure...

1. Je ne sais pas démontrer que la constante de Liouville pour \pi existe et est de l'ordre de 7.6063
2. Même en l'admettant, majorer M_p par p^{\alpha},\;\alpha<0 je ne vois pas...

Conclusion : si tu as des démonstrations sur ces deux points, je suis preneur.

Posté par
astroq123
re : Serie numerique 31-05-17 à 20:03

Merci pour votre aide !!!

Posté par
astroq123
re : Serie numerique 31-05-17 à 20:04

Peut etre la resolution est difficile, mais interessante!



Vous devez être membre accéder à ce service...

Pas encore inscrit ?

1 compte par personne, multi-compte interdit !

Ou identifiez-vous :


Rester sur la page

Inscription gratuite

Fiches en rapport

parmi 1675 fiches de maths

Désolé, votre version d'Internet Explorer est plus que périmée ! Merci de le mettre à jour ou de télécharger Firefox ou Google Chrome pour utiliser le site. Votre ordinateur vous remerciera !